Definite integral giving negative value as a result?Why do I get a negative value for this integral?Solving a...

Why do I get two different answers for this counting problem?

Character reincarnated...as a snail

How can bays and straits be determined in a procedurally generated map?

Doing something right before you need it - expression for this?

How to format long polynomial?

NMaximize is not converging to a solution

Intersection point of 2 lines defined by 2 points each

Languages that we cannot (dis)prove to be Context-Free

Approximately how much travel time was saved by the opening of the Suez Canal in 1869?

"You are your self first supporter", a more proper way to say it

What would happen to a modern skyscraper if it rains micro blackholes?

dbcc cleantable batch size explanation

Is it unprofessional to ask if a job posting on GlassDoor is real?

Cross compiling for RPi - error while loading shared libraries

Replacing matching entries in one column of a file by another column from a different file

Can I make popcorn with any corn?

What's the output of a record needle playing an out-of-speed record

If human space travel is limited by the G force vulnerability, is there a way to counter G forces?

Can a Cauchy sequence converge for one metric while not converging for another?

How does quantile regression compare to logistic regression with the variable split at the quantile?

Watching something be written to a file live with tail

LWC SFDX source push error TypeError: LWC1009: decl.moveTo is not a function

Was any UN Security Council vote triple-vetoed?

Accidentally leaked the solution to an assignment, what to do now? (I'm the prof)



Definite integral giving negative value as a result?


Why do I get a negative value for this integral?Solving a definite integralReal integral giving a complex resultProgression from indefinite integral to definite integral - $int_{0}^{2pi}frac{1}{5-3cos x} dx$Calculation of definite integralWithout calculating the integral decide if integral is positive or negative / which integral is bigger?Definite integral of absolute value function?Variable substitution in definite integralDefinite integral over singularityInner Product, Definite Integral













4












$begingroup$


I want to calculate definite integral



$$int_{-2}^{-1} frac{1}{x^2}e^{frac{1}{x}} dx = Omega$$



$$int frac{1}{x^2}e^{frac{1}{x}} dx=-e^{frac{1}{x}}+C$$



so:



$$Omega = [-e^{frac{1}{-2}}]-[-e^{frac{1}{-1}}]=-frac{1}{sqrt{e}} + frac{1}{e}$$



which is a negative value. I believe it should be positive.



What went wrong in the process?










share|cite|improve this question











$endgroup$








  • 2




    $begingroup$
    How exactly did you go about calculating the antiderivative? Wolfram Alpha gives a much different result.
    $endgroup$
    – Eevee Trainer
    4 hours ago






  • 2




    $begingroup$
    Your antiderivative is completely incorrect: The derivative of $e^{1/x^2}$ is $e^{1/x^2} / (-x^3)$. The red flag that you found is indeed a correct one, and shows that your answer cannot be right. This is a good thing to check.
    $endgroup$
    – T. Bongers
    4 hours ago












  • $begingroup$
    Thanks. I have fixed it now. I meant $int frac{1}{x^2} e^{frac{1}{x}}dx$.
    $endgroup$
    – weno
    4 hours ago








  • 5




    $begingroup$
    You flipped the interval's endpoints. $-2<-1$
    $endgroup$
    – mr_e_man
    4 hours ago










  • $begingroup$
    And just to confirm, taking account of @mr_e_man’s comment above, your work seems correct.
    $endgroup$
    – Lubin
    4 hours ago
















4












$begingroup$


I want to calculate definite integral



$$int_{-2}^{-1} frac{1}{x^2}e^{frac{1}{x}} dx = Omega$$



$$int frac{1}{x^2}e^{frac{1}{x}} dx=-e^{frac{1}{x}}+C$$



so:



$$Omega = [-e^{frac{1}{-2}}]-[-e^{frac{1}{-1}}]=-frac{1}{sqrt{e}} + frac{1}{e}$$



which is a negative value. I believe it should be positive.



What went wrong in the process?










share|cite|improve this question











$endgroup$








  • 2




    $begingroup$
    How exactly did you go about calculating the antiderivative? Wolfram Alpha gives a much different result.
    $endgroup$
    – Eevee Trainer
    4 hours ago






  • 2




    $begingroup$
    Your antiderivative is completely incorrect: The derivative of $e^{1/x^2}$ is $e^{1/x^2} / (-x^3)$. The red flag that you found is indeed a correct one, and shows that your answer cannot be right. This is a good thing to check.
    $endgroup$
    – T. Bongers
    4 hours ago












  • $begingroup$
    Thanks. I have fixed it now. I meant $int frac{1}{x^2} e^{frac{1}{x}}dx$.
    $endgroup$
    – weno
    4 hours ago








  • 5




    $begingroup$
    You flipped the interval's endpoints. $-2<-1$
    $endgroup$
    – mr_e_man
    4 hours ago










  • $begingroup$
    And just to confirm, taking account of @mr_e_man’s comment above, your work seems correct.
    $endgroup$
    – Lubin
    4 hours ago














4












4








4





$begingroup$


I want to calculate definite integral



$$int_{-2}^{-1} frac{1}{x^2}e^{frac{1}{x}} dx = Omega$$



$$int frac{1}{x^2}e^{frac{1}{x}} dx=-e^{frac{1}{x}}+C$$



so:



$$Omega = [-e^{frac{1}{-2}}]-[-e^{frac{1}{-1}}]=-frac{1}{sqrt{e}} + frac{1}{e}$$



which is a negative value. I believe it should be positive.



What went wrong in the process?










share|cite|improve this question











$endgroup$




I want to calculate definite integral



$$int_{-2}^{-1} frac{1}{x^2}e^{frac{1}{x}} dx = Omega$$



$$int frac{1}{x^2}e^{frac{1}{x}} dx=-e^{frac{1}{x}}+C$$



so:



$$Omega = [-e^{frac{1}{-2}}]-[-e^{frac{1}{-1}}]=-frac{1}{sqrt{e}} + frac{1}{e}$$



which is a negative value. I believe it should be positive.



What went wrong in the process?







calculus integration definite-integrals






share|cite|improve this question















share|cite|improve this question













share|cite|improve this question




share|cite|improve this question








edited 4 hours ago









Eevee Trainer

9,92931740




9,92931740










asked 4 hours ago









wenoweno

39611




39611








  • 2




    $begingroup$
    How exactly did you go about calculating the antiderivative? Wolfram Alpha gives a much different result.
    $endgroup$
    – Eevee Trainer
    4 hours ago






  • 2




    $begingroup$
    Your antiderivative is completely incorrect: The derivative of $e^{1/x^2}$ is $e^{1/x^2} / (-x^3)$. The red flag that you found is indeed a correct one, and shows that your answer cannot be right. This is a good thing to check.
    $endgroup$
    – T. Bongers
    4 hours ago












  • $begingroup$
    Thanks. I have fixed it now. I meant $int frac{1}{x^2} e^{frac{1}{x}}dx$.
    $endgroup$
    – weno
    4 hours ago








  • 5




    $begingroup$
    You flipped the interval's endpoints. $-2<-1$
    $endgroup$
    – mr_e_man
    4 hours ago










  • $begingroup$
    And just to confirm, taking account of @mr_e_man’s comment above, your work seems correct.
    $endgroup$
    – Lubin
    4 hours ago














  • 2




    $begingroup$
    How exactly did you go about calculating the antiderivative? Wolfram Alpha gives a much different result.
    $endgroup$
    – Eevee Trainer
    4 hours ago






  • 2




    $begingroup$
    Your antiderivative is completely incorrect: The derivative of $e^{1/x^2}$ is $e^{1/x^2} / (-x^3)$. The red flag that you found is indeed a correct one, and shows that your answer cannot be right. This is a good thing to check.
    $endgroup$
    – T. Bongers
    4 hours ago












  • $begingroup$
    Thanks. I have fixed it now. I meant $int frac{1}{x^2} e^{frac{1}{x}}dx$.
    $endgroup$
    – weno
    4 hours ago








  • 5




    $begingroup$
    You flipped the interval's endpoints. $-2<-1$
    $endgroup$
    – mr_e_man
    4 hours ago










  • $begingroup$
    And just to confirm, taking account of @mr_e_man’s comment above, your work seems correct.
    $endgroup$
    – Lubin
    4 hours ago








2




2




$begingroup$
How exactly did you go about calculating the antiderivative? Wolfram Alpha gives a much different result.
$endgroup$
– Eevee Trainer
4 hours ago




$begingroup$
How exactly did you go about calculating the antiderivative? Wolfram Alpha gives a much different result.
$endgroup$
– Eevee Trainer
4 hours ago




2




2




$begingroup$
Your antiderivative is completely incorrect: The derivative of $e^{1/x^2}$ is $e^{1/x^2} / (-x^3)$. The red flag that you found is indeed a correct one, and shows that your answer cannot be right. This is a good thing to check.
$endgroup$
– T. Bongers
4 hours ago






$begingroup$
Your antiderivative is completely incorrect: The derivative of $e^{1/x^2}$ is $e^{1/x^2} / (-x^3)$. The red flag that you found is indeed a correct one, and shows that your answer cannot be right. This is a good thing to check.
$endgroup$
– T. Bongers
4 hours ago














$begingroup$
Thanks. I have fixed it now. I meant $int frac{1}{x^2} e^{frac{1}{x}}dx$.
$endgroup$
– weno
4 hours ago






$begingroup$
Thanks. I have fixed it now. I meant $int frac{1}{x^2} e^{frac{1}{x}}dx$.
$endgroup$
– weno
4 hours ago






5




5




$begingroup$
You flipped the interval's endpoints. $-2<-1$
$endgroup$
– mr_e_man
4 hours ago




$begingroup$
You flipped the interval's endpoints. $-2<-1$
$endgroup$
– mr_e_man
4 hours ago












$begingroup$
And just to confirm, taking account of @mr_e_man’s comment above, your work seems correct.
$endgroup$
– Lubin
4 hours ago




$begingroup$
And just to confirm, taking account of @mr_e_man’s comment above, your work seems correct.
$endgroup$
– Lubin
4 hours ago










1 Answer
1






active

oldest

votes


















4












$begingroup$

What you effectively did was swap the order of evaluation for the fundamental theorem of calculus. Recall:



$$int_a^b f(x)dx = F(b) - F(a)$$



when the antiderivative of $f$ is $F$. You instead have $F(a) - F(b)$ ($a=-2,b=-1$) in this case. The end result is merely a sign error - you have precisely the negative of the answer which you should expect.






share|cite|improve this answer









$endgroup$














    Your Answer





    StackExchange.ifUsing("editor", function () {
    return StackExchange.using("mathjaxEditing", function () {
    StackExchange.MarkdownEditor.creationCallbacks.add(function (editor, postfix) {
    StackExchange.mathjaxEditing.prepareWmdForMathJax(editor, postfix, [["$", "$"], ["\\(","\\)"]]);
    });
    });
    }, "mathjax-editing");

    StackExchange.ready(function() {
    var channelOptions = {
    tags: "".split(" "),
    id: "69"
    };
    initTagRenderer("".split(" "), "".split(" "), channelOptions);

    StackExchange.using("externalEditor", function() {
    // Have to fire editor after snippets, if snippets enabled
    if (StackExchange.settings.snippets.snippetsEnabled) {
    StackExchange.using("snippets", function() {
    createEditor();
    });
    }
    else {
    createEditor();
    }
    });

    function createEditor() {
    StackExchange.prepareEditor({
    heartbeatType: 'answer',
    autoActivateHeartbeat: false,
    convertImagesToLinks: true,
    noModals: true,
    showLowRepImageUploadWarning: true,
    reputationToPostImages: 10,
    bindNavPrevention: true,
    postfix: "",
    imageUploader: {
    brandingHtml: "Powered by u003ca class="icon-imgur-white" href="https://imgur.com/"u003eu003c/au003e",
    contentPolicyHtml: "User contributions licensed under u003ca href="https://creativecommons.org/licenses/by-sa/3.0/"u003ecc by-sa 3.0 with attribution requiredu003c/au003e u003ca href="https://stackoverflow.com/legal/content-policy"u003e(content policy)u003c/au003e",
    allowUrls: true
    },
    noCode: true, onDemand: true,
    discardSelector: ".discard-answer"
    ,immediatelyShowMarkdownHelp:true
    });


    }
    });














    draft saved

    draft discarded


















    StackExchange.ready(
    function () {
    StackExchange.openid.initPostLogin('.new-post-login', 'https%3a%2f%2fmath.stackexchange.com%2fquestions%2f3176540%2fdefinite-integral-giving-negative-value-as-a-result%23new-answer', 'question_page');
    }
    );

    Post as a guest















    Required, but never shown

























    1 Answer
    1






    active

    oldest

    votes








    1 Answer
    1






    active

    oldest

    votes









    active

    oldest

    votes






    active

    oldest

    votes









    4












    $begingroup$

    What you effectively did was swap the order of evaluation for the fundamental theorem of calculus. Recall:



    $$int_a^b f(x)dx = F(b) - F(a)$$



    when the antiderivative of $f$ is $F$. You instead have $F(a) - F(b)$ ($a=-2,b=-1$) in this case. The end result is merely a sign error - you have precisely the negative of the answer which you should expect.






    share|cite|improve this answer









    $endgroup$


















      4












      $begingroup$

      What you effectively did was swap the order of evaluation for the fundamental theorem of calculus. Recall:



      $$int_a^b f(x)dx = F(b) - F(a)$$



      when the antiderivative of $f$ is $F$. You instead have $F(a) - F(b)$ ($a=-2,b=-1$) in this case. The end result is merely a sign error - you have precisely the negative of the answer which you should expect.






      share|cite|improve this answer









      $endgroup$
















        4












        4








        4





        $begingroup$

        What you effectively did was swap the order of evaluation for the fundamental theorem of calculus. Recall:



        $$int_a^b f(x)dx = F(b) - F(a)$$



        when the antiderivative of $f$ is $F$. You instead have $F(a) - F(b)$ ($a=-2,b=-1$) in this case. The end result is merely a sign error - you have precisely the negative of the answer which you should expect.






        share|cite|improve this answer









        $endgroup$



        What you effectively did was swap the order of evaluation for the fundamental theorem of calculus. Recall:



        $$int_a^b f(x)dx = F(b) - F(a)$$



        when the antiderivative of $f$ is $F$. You instead have $F(a) - F(b)$ ($a=-2,b=-1$) in this case. The end result is merely a sign error - you have precisely the negative of the answer which you should expect.







        share|cite|improve this answer












        share|cite|improve this answer



        share|cite|improve this answer










        answered 4 hours ago









        Eevee TrainerEevee Trainer

        9,92931740




        9,92931740






























            draft saved

            draft discarded




















































            Thanks for contributing an answer to Mathematics Stack Exchange!


            • Please be sure to answer the question. Provide details and share your research!

            But avoid



            • Asking for help, clarification, or responding to other answers.

            • Making statements based on opinion; back them up with references or personal experience.


            Use MathJax to format equations. MathJax reference.


            To learn more, see our tips on writing great answers.




            draft saved


            draft discarded














            StackExchange.ready(
            function () {
            StackExchange.openid.initPostLogin('.new-post-login', 'https%3a%2f%2fmath.stackexchange.com%2fquestions%2f3176540%2fdefinite-integral-giving-negative-value-as-a-result%23new-answer', 'question_page');
            }
            );

            Post as a guest















            Required, but never shown





















































            Required, but never shown














            Required, but never shown












            Required, but never shown







            Required, but never shown

































            Required, but never shown














            Required, but never shown












            Required, but never shown







            Required, but never shown







            Popular posts from this blog

            Gersau Kjelder | Navigasjonsmeny46°59′0″N 8°31′0″E46°59′0″N...

            What is the “three and three hundred thousand syndrome”?Who wrote the book Arena?What five creatures were...

            Are all UTXOs locked by an address spent in a transaction?UTXO all sent to change address?Signing...